Last visit was: 25 Apr 2024, 00:11 It is currently 25 Apr 2024, 00:11

Close
GMAT Club Daily Prep
Thank you for using the timer - this advanced tool can estimate your performance and suggest more practice questions. We have subscribed you to Daily Prep Questions via email.

Customized
for You

we will pick new questions that match your level based on your Timer History

Track
Your Progress

every week, we’ll send you an estimated GMAT score based on your performance

Practice
Pays

we will pick new questions that match your level based on your Timer History
Not interested in getting valuable practice questions and articles delivered to your email? No problem, unsubscribe here.
Close
Request Expert Reply
Confirm Cancel
SORT BY:
Kudos
Math Expert
Joined: 02 Sep 2009
Posts: 92901
Own Kudos [?]: 618866 [31]
Given Kudos: 81588
Send PM
Most Helpful Reply
GMAT Club Legend
GMAT Club Legend
Joined: 12 Sep 2015
Posts: 6821
Own Kudos [?]: 29918 [8]
Given Kudos: 799
Location: Canada
Send PM
General Discussion
Intern
Intern
Joined: 12 Mar 2018
Posts: 37
Own Kudos [?]: 38 [3]
Given Kudos: 105
Location: Iran (Islamic Republic of)
GPA: 3.64
WE:Management Consulting (Energy and Utilities)
Send PM
VP
VP
Joined: 11 Aug 2020
Posts: 1262
Own Kudos [?]: 201 [1]
Given Kudos: 332
Send PM
Re: If x, y, and z satisfy the inequalities shown, what is the least possi [#permalink]
1
Bookmarks
Use the answer choices to cheat. We see that there are no negative values so that makes our lives easier. Look to see whether 0 is obtainable given the ranges. It is.

x = -6
y = 2
z = 4

x + y + z = -6 + 2 + 4 = 0

A.
Manhattan Prep Instructor
Joined: 22 Mar 2011
Posts: 2642
Own Kudos [?]: 7775 [1]
Given Kudos: 55
GMAT 2: 780  Q50  V50
Send PM
Re: If x, y, and z satisfy the inequalities shown, what is the least possi [#permalink]
1
Kudos
Expert Reply
VeeB wrote:
From the answer choices we see the lowest possible is 0 which
So is |(-5)+2+3| = 0 also a right way to arrive?

Posted from my mobile device

­No, because x can't = -5. Its absolute value must be between 6 and 8 inclusive, so if it's negative it must be between -8 and -6. 
GMAT Club Legend
GMAT Club Legend
Joined: 18 Aug 2017
Status:You learn more from failure than from success.
Posts: 8019
Own Kudos [?]: 4096 [0]
Given Kudos: 242
Location: India
Concentration: Sustainability, Marketing
GMAT Focus 1:
545 Q79 V79 DI73
GPA: 4
WE:Marketing (Energy and Utilities)
Send PM
Re: If x, y, and z satisfy the inequalities shown, what is the least possi [#permalink]
Bunuel wrote:
6 ≤ |x| ≤ 8

1 ≤ |y| ≤2

3 ≤ |z| ≤ 4

If x, y, and z satisfy the inequalities shown, what is the least possible value of |x+y+z| ?

A. 0
B. 1
C. 2
D. 3
E. 4



GMATinsight :

Sir how to solve this question ?

least values for x can be 6 , y=1 and z 3 ,
Is there a (-) sign missing from the given expressions?
Intern
Intern
Joined: 15 Dec 2023
Posts: 29
Own Kudos [?]: 19 [0]
Given Kudos: 46
Location: India
Send PM
Re: If x, y, and z satisfy the inequalities shown, what is the least possi [#permalink]
From the answer choices we see the lowest possible is 0 which
So is |(-5)+2+3| = 0 also a right way to arrive?

Posted from my mobile device
GMAT Club Bot
Re: If x, y, and z satisfy the inequalities shown, what is the least possi [#permalink]
Moderators:
Math Expert
92900 posts
Senior Moderator - Masters Forum
3137 posts

Powered by phpBB © phpBB Group | Emoji artwork provided by EmojiOne